2
$\begingroup$

Consider the power sum $$S_a(b)=1^{2b}+2^{2b}+\cdots+(3a-2)^{2b}.$$ Let $\nu_3(x)$ denote the $3$-adic valuation of $x$.

QUESTION 1. (milder) Is this true? $$\nu_3\left(\frac{S_a(b)}{S_a(1)}\right)=0.$$ QUESTION 2. Is this true? $\nu_3(S_a(b))=\nu_3(2a-1)$.

$\endgroup$
5
  • 3
    $\begingroup$ Questions 1 and 2 are clearly equivalent, since $S_a\left(1\right) = 1^2 + 2^2 + \cdots + \left(3a-2\right)^2 = \dfrac12 \left(2a-1\right) \left(3a-1\right) \left(3a-2\right)$ has the same $3$-adic valuation as $2a-1$. $\endgroup$ Jul 9, 2021 at 21:59
  • 1
    $\begingroup$ I suspect that your conjecture is indeed true, and in fact that $S_a\left(b\right)$ is a polynomial in $a$ of the form $\dfrac{1}{m} \left(2a-1\right) \left(3 q\left(a\right) + r\right)$, where $q$ is an integral polynomial and where $m$ and $r$ are two integers not divisible by $3$. This should really be a property of Faulhaber polynomials, but I am too faul to translate it into that language :) (I have checked it for all $b \leq 8$.) $\endgroup$ Jul 9, 2021 at 22:02
  • $\begingroup$ @darijgrinberg: interesting. How do you ensure that $m$ and $r$ are not divisible by $3$? $\endgroup$ Jul 10, 2021 at 17:15
  • $\begingroup$ That's why I'm saying "suspect"... $\endgroup$ Jul 10, 2021 at 17:43
  • $\begingroup$ Yes, you did say that. :-) $\endgroup$ Jul 10, 2021 at 17:46

1 Answer 1

6
$\begingroup$

Notice that $$2S_a(b) \equiv 1^{2b} + 2^{2b} + \dots + (6a-4)^{2b} \pmod{6a-3}.$$

From Faulhaber's formula, we have $$1^{2b} + 2^{2b} + \dots + (6a-4)^{2b} \equiv B_{2b} (6a-3) \pmod{6a-3}.$$ It follows that $$S_a(b) \equiv \frac32B_{2b} (2a-1)\pmod{3(2a-1)},$$ where $\nu_3(\frac32 B_{2b})=0$ by von Staudt–Clausen theorem. Hence, $\nu_3(S_a(b))=\nu_3(2a-1)$.


ADDED. Explanation of the Faulhaber's formula implication:

Lemma. Let $n,m$ be positive integers, $m$ is even. Then $$1^{m} + 2^{m} + \dots + (n-1)^{m} \equiv B_{m} n \pmod{n}.$$

Proof. For $m=2$, the statement can be verified directly: $$1^2 + 2^2 + \dots + (n-1)^2 = \frac{(n-1)n(2n-1)}{6} \equiv B_2 n\pmod{n}.$$ For the rest assume $m\geq 4$.

Using Faulhaber's formula and taking into account that $B_{m-1}=0$, we have \begin{split} & 1^{m} + 2^{m} + \dots + (n-1)^{m} \equiv 1^{m} + 2^{m} + \dots + n^{m} \\ &\equiv B_{m} n + \frac{1}{m+1} \sum_{k=3}^{m+1} \binom{m+1}{k} B_{m+1-k} n^k \pmod{n}. \end{split} It remains to show that $\frac{1}{m+1} \binom{m+1}{k} B_{m+1-k} n^k \equiv 0\pmod{n}$ for any integer $k\in [3,m+1]$.

Consider any prime $p\mid n$, and let $t:=\nu_p(n)\geq 1$. Our goal is to show that $\nu_p\big(\frac{1}{m+1} \binom{m+1}{k} B_{m+1-k}\big) + t(k-1) \geq 0$. It is enough to focus on the case $t=1$, from which the case $t>1$ follows instantly.

Since by von Staudt–Clausen theorem the denominators of Bernoulli numbers are square-free, we need to show that $$\nu_p\big(\frac{1}{m+1} \binom{m+1}{k}\big) + k - 2 \geq 0.$$

Noticing that $\nu_p\big(\frac{1}{m+1} \binom{m+1}{k}\big) = \nu_p\big(\frac1k\binom{m}{k-1}\big)\geq -\nu_p(k)$, our goal reduces to showing that $$k-2-\nu_p(k)\geq 0.$$ For $k=3$ and $k=4$, this inequality is trivial, while for $k\geq 5$ it follows from the bound $\nu_p(k)\leq \log_2(k)$. QED

$\endgroup$
2
  • $\begingroup$ Would it be possible to elaborate/expand your 2nd congruence with Faulhaber? $\endgroup$ Jul 10, 2021 at 14:41
  • $\begingroup$ @T.Amdeberhan: I've just added an explanation. $\endgroup$ Jul 11, 2021 at 17:09

Your Answer

By clicking “Post Your Answer”, you agree to our terms of service and acknowledge that you have read and understand our privacy policy and code of conduct.

Not the answer you're looking for? Browse other questions tagged or ask your own question.